Candice works at a grocery store. She works 35 hours a week and
gets paid $8.50 an hour. For every hour over 35, Candice gets
paid $10 an hour. If she was paid $357.50 last week, how many
hours over 35 did Candice work?

Answers

Answer 1

9514 1404 393

Answer:

 6

Step-by-step explanation:

For 35 hours, Candice is paid ...

  ($8.50/h)(35 h) = $297.50

Any amount in excess of that is paid at $10 per hour. The amount of excess pay is ...

  $357.50 -297.50 = $60.00

Then the number of overtime hours was ...

  $60/($10/h) = 6 h

Candice worked 6 hours over 35 last week.


Related Questions

Jason works as a delivery boy. He is paid daily, and the amount he is paid depends on the number of deliveries he makes. Jason’s earnings per day, in dollars, for 14 workdays are below.

Answers

A student earns a day to deliverer a newspaper= $25
She delivers for d days .
So she earns in d days = $25d
She also earns extra charges for each newspaper = $0.75 =
If she delivers n newspapers .
So. she earns extra charges for n newspapers =
Now we are also given that She earns $5 for each new customer .
If she has c new customers .
So, she earns for c customers = $5c
So, her total earning =
Let T denotes the total earning .
So,
Thus Option 2 is correct.

If you take a class and have three marking periods in the class and in the first period your average grade was a 85.56 the second marking period your average was a 60.57 and then for the final marking period your average was a 63.63 what will be your overall average?

Answers

69.92 when you add them all up and divide it you get 69.92

What did she do wrong ?

Answers

Answer:

She wrongly added the equations

Step-by-step explanation:

Given

[tex]-3x + y = 8[/tex]

[tex]-3x + y = -4[/tex]

Required

Her mistake

The mistake is when she added the equations.

When both equations are added, the result is:

[tex]-3x -3x + y+y=8-4[/tex]

[tex]-6x +2y=4[/tex]

and not

[tex]2y = 4[/tex]

The suggestion to avoid such mistake is for the student to check the appropriate signs of each term before adding/subtracting.

PLEASE HELP ASAP PLSSSS !!!

Answers

x=22/y=6
Step by step below

PLSSSSS HELP!!!!!!!!!!

Answers

Answer:

3x-26=2x(because they are vertical angle)

or,3x-2x=26

therefore x=26

Answer:

Step-by-step explanation:

2x + (3x-26) = 90

5x = 90+26

5x = 116

x = 116/5

x = 23.2

A cylindrical container has a diameter of 14 cm and a height of 10 cm, as
shown in the diagram. What is the volume of this container to the nearest
tenth of a cubic centimeter?*
14cm
10cm
1539.4
6157.5
490.0
219.9

Answers

Answer:

1539.4

Step-by-step explanation:

Steps:

Volume of a cylinder= πr²h

radius= diameter/2

         = 14/2

         = 7 cm

Volume= π x 7² x 10

            = 1539.38

            = 1539.4 cm³

Answer:

1539.4

Step-by-step explanation:

to find the area of a cylinder, you solve pi x radius²

3.14 x 7² = 153.94

then you multiply times the height

153.94 x 10 = 1549.4

thvuvugufugy i need help pls i beg​

Answers

Answer:

A-10

B- -12

C-3.6

If you cant understand B is -12

Please answer this will give brainliest

Answers

Answer:

A. -8

B. 23

C. 3

D. 5

Step-by-step explanation:

5th grade math. correct answer will be marked brainliest

Answers

Answer:

31 and a half

Step-by-step explanation:

not sure tho_____________

Answer:

31.5 is ur answer

Step-by-step explanation:

so what you have to do is add up 5.5 + 5.5 + 6+ 7 +7.5 which gives u 31.5

11. Find the sum of the arithmetic series. overset [22] underset{k=1}\huge{Sigma) (-4k - 13) =​

Answers

Answer:

okay, we need to add up for all k=1 till k=22

we could do it like this

(-4*1 -13) + (-4*2 -13) + (-4*3 -13) + ...

but this is long and boring, so we need another valid, yet vastly faster method to calculate this sum

we know that the "-13" part will occur 22 times, so let's write this component as "-13*22"

the -4k gents bigger with each step, k will take all the natural numbers from 1 to 22

so we could write

"-4 * (1+2+3+4+5+6+7...+21+22l

alot simpler and faster, but not fast enough

let's add up all numbers from 1 to 22 into one number

1 + 22 = 23

2 + 21 = 23

3 + 20 = 23

...

this works eleven times (bc we use up 2 numbers in each step), so the numbers from 1 to 22 added up are just

23*11= 230+23 = 252

now let's construct the final calculation

-4 * 252 -13*22 = -1294

there's your sum.

hope it helps you overall.

brainliest would be very kind ic some else leaves an answer, may it be just a greeting

f(x) = x ^ 2 . What is g(x) ?

Answers

Given:

The function is:

[tex]f(x)=x^2[/tex]

The graphs of functions [tex]f(x)[/tex] and [tex]g(x)[/tex].

To find:

The function [tex]g(x)[/tex].

Solution:

We have,

[tex]f(x)=x^2[/tex]

The function [tex]f(x)[/tex] is vertically compresses to get the graph of the function [tex]g(x)[/tex]. So, the function [tex]g(x)[/tex] is:

[tex]g(x)=kf(x)[/tex]

[tex]g(x)=kx^2[/tex]               ...(i)

From the given graph it is clear that the graph of the function [tex]g(x)[/tex] passes through the point (3,3). So, putting [tex]x=3[/tex] and [tex]g(x)=3[/tex] in the above function, we get

[tex]3=k(3)^2[/tex]

[tex]3=9k[/tex]

[tex]\dfrac{3}{9}=k[/tex]

[tex]\dfrac{1}{3}=k[/tex]

Putting [tex]k=\dfrac{1}{3}[/tex] in (i), we get

[tex]g(x)=\dfrac{1}{3}x^2[/tex]

Therefore, the correct option is D.

URGENT HELP! Graph and indicate the 3rd Critical Point

Answers

6 is the critical point

which is equivalent to (2-3i)(i³⁰³)?

Answers

Answer:

-2i - 3

I can't see a simpler way to explain it. You just have to work your way through each step.

Step-by-step explanation:

The question is what is i^303?

Remember that when the power is divisible by 4 the expression changes to 1

i^1 = i

i^2 = i * i = sqrt(-1)*sqrt(-1) = - 1

i^3 = i^2 * i = - i

i^4 = i^2 * i^2 = -1 * - 1 = 1

So what does i^303 =

i^300 * i^3

But 300 is evenly divisible by 4. (300 / 4 = 75) so i^300 = 1^75

i^3 = -i

So now you are ready to do the question

(2 - 3i) * -i

2*-i - 3i(-i)

-2i - 3(i^2 * - 1)

-2i - 3(-1 * - 1)

-2i - 3(1)

-2i - 3

the product of n and 6 divided by 4 in algebraic expression​

Answers

Answer:

[tex]6n \div 4[/tex]

Step-by-step explanation:

n is a variable and multiplied by 6, so it is attached to 6. (6 is known as the coefficient.) Then it is divided by 4. (4 is known as the constant because there is no variable attached to it.)

[tex]6n \div 4[/tex]

f(x) = -2 ( 1 - 1/4 x ) compute f( -3)

Answers

Answer:

-3 1/2 or -3.5

Step-by-step explanation:

* means multiply

just put 3 where the x is

so

f(x) = -2 ( 1 - 1/4 x )

f( -3) = -2 ( 1 - 1/4 * -3 )

f( -3) = -2 ( 1 3/4 )

f( -3) = - 3 1/2

A bag of candy has equal numbers of candies in eight colors: blue, red,
brown, green, yellow, orange, pink, and black. If you eat them one by one,
what's the probability of getting your first red candy on or before the fifth
pick?

Answers

Answer:

the answer would be b I'm pretty sure

If 30 men can complete a work in 40 days,
In how many days 15 men will complete
it?

Answers

Answer:

80

Step-by-step explanation:

djdjdjdjdjdjkkkdkjrr

80 days because you have half the men so you need to double the number of days to make up for the missing men

24+67-67+78/2+6-89-4​

Answers

Answer:

1. 2 is the correct answer

Step-by-step explanation:

24+67-67+78 = 102

102 /2+6-89-4​

= 1. 2

I am sure about my answer , trust me.

Hope this answer helps you :)

Have a great day

Mark brainliest

The temperature at 8 a.m. was -6°C.
By noon, the temperature was 3°C.
Find the change in temperature.
11

Answers

Answer:

9

Step-by-step explanation:

3 - ( -6 ) = 9

Hope this helps! Brainliest?

Answer:

9

Step-by-step explanation:

the other answer is right.

to add a little bit of explanation :

you need to calculate the difference between -6 and +3.

and we are looking on the number line only for a distance and not for a direction, as the distance (or simple difference) between -6 and +3 is the same as between +3 and -6.

so, whatever we get as difference, we need to treat that result as positive number.

so, we get the difference between two numbers by subtracting one from the other.

either 3 - (-6), or -6 - 3

you do know that subtracting a negative number is the same as adding the same positive number ?

e.g. 3 - (-6) = 3 + 6 = 9

as for the other direction -6 - 3 = -9

but again, we have to take this in this context as a positive number.

in math we call this the absolute value of a number.

so, the answer is always 9.

What is the area? Round to the nearest tenth

Answers

Answer:

60.8 mm^2

D

Step-by-step explanation:

Remark

You have to assume that the left and right sides are both 9.3. The question is undoable without that assumption.

Next, you have to assume that The top is 7.8 mm across and both ends meet 9.3 at right angles. Again if that is not true, the problem can't be done. Sometimes people making these questions make errors which you are asked to correct.

Solution

So let's assume that everything I've assumed is correct.

Find the area of the rectangle.

Area = L*W

L = 9.3

W = 7.8

Area = 7.8 * 9.3

Area = 72.54        Do your rounding at the end.

Now find the are of the triangle that has been cut out.

The height = 3

The base = 7.8

Area = 1/2 b * h

Area = 1/2 7.8 * 3

Area = 11.7

The area of the figure = area of the rectangle - the triangle's area

Figure Area = 72.54 - 11.7

Figure Area = 60.84

Will Mark Brainlest!!!!!! pls helppo meee​

Answers

Answer:

Step-by-step explanation:

44%adults favor the use of unmanned drones by police agencies. Twelve U.S. adults are randomly selected. Find the probability that the number of U.S. adults who favor the use of unmanned drones by police agencies is​ (a) exactly​ three, (b) at least​ four, (c) less than eight.

Answers

Answer:

.101501615

.84982392

.901244701

Step-by-step explanation:

For this question use a binomial distribtuion

a.

12C3*.44³*(1-.44)⁹= .101501615

b.

to find at least 4 find the probability of less than 4 and take its compliment

12C0*(1-.44)¹²+12C1*.44*(1-.44)¹¹+12C2*.44²*(1-.44)¹⁰+12C3*.44³*(1-.44)⁹= .15017608

1-.15017608=.84982392

c.

To find less than eight's probability find the proability of at least 8 and take its compliment

12C8*.44⁸(1-.44)⁴+12C9*.44⁹(1-.44)³+12C10*.44¹⁰(1-.44)²+12C11*.44¹¹*(1-.44)+12C12*.44¹²= .098755299

1-.098755299= .901244701

Jessica left her running shoes at school yesterday. Today she walked 4 miles to school to get her shoes, she ran home along the same route, and the total time for both trips was 2 hours. Jessica walked and ran at constant speeds, and she ran 3 miles per hour faster than she walked.



What was Jessica's walking speed in miles per hour?

Answers

Jessica left her running shoes at school yesterday. Today she walked 4 miles to school to get her shoes, she ran home along the same route, and the total time for both trips was 2 hours. Jessica walked and ran at constant speeds, and she ran 3 miles per hour faster than she walked.

What was Jessica's walking speed in miles per hour?

Can you help and do it step by step

Answers

Answer:

$4800

Step-by-step explanation:

SI=PRT DIVIDED BY 100..

So it is going to be 400×2×6 divided by 100 which will be 4800

Compound interest . A student get a grant of $10000 a year assuming her grant is increased by 7% each year what will her grant be in four years time

Answers

Answer:

Her grant in four years time will be of $13108.

Step-by-step explanation:

Exponential equation:

An exponential equation has the following format:

[tex]A(t) = A(0)(1+r)^t[/tex]

In which A(0) is the initial value and r is the growth rate, as a decimal.

A student get a grant of $10000 a year assuming her grant is increased by 7% each year

This means that [tex]A(0) = 10000, r = 0.07[/tex]

So

[tex]A(t) = A(0)(1+r)^t[/tex]

[tex]A(t) = 10000(1+0.07)^t[/tex]

[tex]A(t) = 10000(1.07)^t[/tex]

What will her grant be in four years time?

This is A(4). So

[tex]A(t) = 10000(1.07)^t[/tex]

[tex]A(4) = 10000(1.07)^4 = 13108[/tex]

Her grant in four years time will be of $13108.


Solve 11ax + 12 ax = 13 ax + 60 for x. Assume a=0.
Α. α = 10a
B. x = 10a
C. x = 6a
D. x =6/a

Answers

Answer:

In the problem, the value of x is equal to 6a.

Step-by-step explanation:

For this equation, we are to solve for x.  

11ax + 12ax = 13ax + 60

Add together 11 and 12 because they have like terms.

23ax = 13ax + 60

Substract 13ax on both sides.

10ax = 60

Now, we divide by 10 on both sides.

ax = 6

In order to see what x is, divide a on both sides.

x = 6a

A theater has 56 rows of seats. If there are 13 seats in the first row, 18 in the second row, 23 in thethird row. How many seats are there in total? Show explain all work.

Answers

Answer:

There are 54 in total because if you add them all it gets you 54

a plumber charges $50 for every house call and an additional $70 for each hour of work for one house call yesterday the plumber earned $435 which equation can be solved to find the number of hours the plumber word on the house call

Answers

Answer:

5.5 hours

Step-by-step explanation:

Since every house has an initial charge of $50, we can begin with that

($50)

Then, let's define h as the number of hours. We know every hour is $70, although the hours is yet unknown so that can be

($70 * h) or simply ($70h)

Now, we know the total should end up to be $435, so after PEMDAS the equation will be

($70h + $50 = $435).

We subtract 50 from both sides so now we have $70h = $385.

Now we divide both sides by 70, giving us h = 5.5. Hope it helped!

The number of hours that the plumber work on the house calls should be 5.5.

Given that,

The charge for a plumber for every house call is $50.And, the extra $70 for each hour. The earning of the plumber is $435.Here we assume the number of hours be h.

Based on the above information, the equation is as follows:

70h + 50 = 435

70h = 385

h = 5.5 hours

Therefore we can conclude that the number of hours that the plumber work on the house calls should be 5.5.

Learn more: brainly.com/question/3530056

2^24 x 3^2 x 1 hhhhhhhhhhhhhhhhhh

Answers

Answer:

The answer is 5184. Hope it helps luv

A tank with capacity of 600 gal of water originally contains 200 gal of water with 100 lb of salt in solution. Water containing 1 lb of salt per gallon is entering at a rate of 3 gal/min, and the mixture is allowed to flow out of the tank at a rate of

Answers

Answer:

Step-by-step explanation:

From the information given:

Tank's Capacity = 600 gal

Original content of water = 200 gal

Salt solution = 100 lb

In the tank:

Suppose x(t) = amount of salt at time (t); &

V(t) = volume of water

Then,

V(t) = 200 + t

and [tex]x'(t) = 3 - \dfrac{2x(t)}{200+t}[/tex]

From the above linear equation, the integrating factor can be computed as:

[tex]x(t) = \Bigg(e^{\int \dfrac{2}{200+t}} \Bigg)^{dt}[/tex]

[tex]= e^{2 \ log (200+t)}[/tex]

= (200 + t)²

The general solution can now be expressed as:

[tex]x(t) = \dfrac{1}{(200+t)^2}\Bigg( \int 3(200+t)^2 \ dt +C\Bigg)[/tex]

We know that C = integrating factor, thus taking the integral:

[tex]x(t) = \dfrac{(200+t)^3 +C}{(200+t)^2}[/tex]

At the initial condition, x(0) = 100

[tex]100= \dfrac{(200)^3 +C}{(200)^2}[/tex]

C = ((200)²×100) - 200³

C = -4 × 10⁶

Hence, at any time t, the amount of salt is:

[tex]x(t) = \dfrac{(200+t^2) - 4\times 10^6}{(200+t)^2}[/tex]

Other Questions
what is the intellect of west and east africa After which Cold War event did the United States and the Soviet Union enter a nuclear arms race? how do i find love brainlist for best (and dont say be yoru self) En un sistema masa resorte (masa atada a un muelle) en qu punto la masa alcanza su mxima velocidad? What is the sum of the polynomials? (6x+7+x2)+(2x2-3) -x^2 +6x+4, 3x^2 +6x+4, 9x+4, 9x^2 +4 In a solution containing 21.2 g sodium carbonate in 1.50 L of solution, calculatethe concentration of the sodium and carbonate ions. Question 7Robert Kennedy tried to help African Americans by Find the circumference of each circle from the given radius or diameterPlease help Helpmeeeeeeeeeeeeeeeeeeeeeee SCIENCEWhich of the following is an example of a glacial formation? (Dont put a random answer or spam things or I will report you and you will lose the points)A. Photosynthesis B. Methane degradation C. PhotolysisD. Weathering of rocks Math help please urgent Lines J and k intersect at point Q and h is a straight line. What is the value of X ? A card deck for a board game has 20 cards, of which 4 are red, 6 are blue, and 10 are purple.What is the probability of randomly selecting a blue card, then a purple card, without replacement? a drop of oil of volume 10^-10 cubic metre spreads out on water to make a circular film of radius 10^-1 metre. what is the thickness of the film? PLEASE HELP!! The green line below *blank*. Check all that apply.A. is parallel to the x-axisB. crosses the originC. is in the xy-planeD. crosses the y-axis Here is a pronoun from a comina in the book of my own. Find the right spot for the best in the bank. Review your story and read it again before submission. 5. Which of the following statements is not true about road workers?A. Road workers try hard to warn you of their presence.B. The lives of road workers are in your hands.C. Road workers use yellow signs, flags, and jackets.D. All statements are true. Find the resultant of these two forces : 2.00 x 10^2 N due east and 4.00 x 10^2 N 30.0 degree north of west PLEASE HELP ASAP PLSSSSS!!! A track coach records the number of miles that he ran per day over the month. The stem-and-leaf plot below summarizes this data.